Please Help

If p is prime number and a and b are natural numbers such that a>b, Prove that

(papb)(ab)modp\large{\dbinom{pa}{pb} \equiv \dbinom{a}{b} \mod p}

#NumberTheory

Note by Md Zuhair
3 years, 8 months ago

No vote yet
1 vote

  Easy Math Editor

This discussion board is a place to discuss our Daily Challenges and the math and science related to those challenges. Explanations are more than just a solution — they should explain the steps and thinking strategies that you used to obtain the solution. Comments should further the discussion of math and science.

When posting on Brilliant:

  • Use the emojis to react to an explanation, whether you're congratulating a job well done , or just really confused .
  • Ask specific questions about the challenge or the steps in somebody's explanation. Well-posed questions can add a lot to the discussion, but posting "I don't understand!" doesn't help anyone.
  • Try to contribute something new to the discussion, whether it is an extension, generalization or other idea related to the challenge.
  • Stay on topic — we're all here to learn more about math and science, not to hear about your favorite get-rich-quick scheme or current world events.

MarkdownAppears as
*italics* or _italics_ italics
**bold** or __bold__ bold

- bulleted
- list

  • bulleted
  • list

1. numbered
2. list

  1. numbered
  2. list
Note: you must add a full line of space before and after lists for them to show up correctly
paragraph 1

paragraph 2

paragraph 1

paragraph 2

[example link](https://brilliant.org)example link
> This is a quote
This is a quote
    # I indented these lines
    # 4 spaces, and now they show
    # up as a code block.

    print "hello world"
# I indented these lines
# 4 spaces, and now they show
# up as a code block.

print "hello world"
MathAppears as
Remember to wrap math in \( ... \) or \[ ... \] to ensure proper formatting.
2 \times 3 2×3 2 \times 3
2^{34} 234 2^{34}
a_{i-1} ai1 a_{i-1}
\frac{2}{3} 23 \frac{2}{3}
\sqrt{2} 2 \sqrt{2}
\sum_{i=1}^3 i=13 \sum_{i=1}^3
\sin \theta sinθ \sin \theta
\boxed{123} 123 \boxed{123}

Comments

Well, (papb)=(pa)!(pb)!(papb)!,\binom{pa}{pb} = \frac{(pa)!}{(pb)!(pa-pb)!}, and now look at the terms on the top and bottom that are divisible by p.p. These are (pa)(p(a1))(p(a2))()(p1)(pb)(p(b1))()(p1)(p(ab))(p(ab1))()(p1)=paa!pbb!pab(ab)!=a!b!(ab)!=(ab). \frac{(pa)(p(a-1))(p(a-2))(\cdots)(p\cdot 1)}{(pb)(p(b-1))(\cdots)(p\cdot 1)(p(a-b))(p(a-b-1))(\cdots)(p\cdot 1)} = \frac{p^a a!}{p^b b! \cdot p^{a-b} (a-b)!} = \frac{a!}{b!(a-b)!} = \binom{a}{b}. What are the leftover terms mod pp? It's just a bunch of copies of (p1)!(p-1)!: (p1)!a(p1)!b(p1)!ab1. \frac{(p-1)!^a}{(p-1)!^b(p-1)!^{a-b}} \equiv 1. So that'll do it.

You might be interested in the Lucas' theorem wiki.

Patrick Corn - 3 years, 8 months ago

Log in to reply

This is a lemma in the proof of lucas' theorem.

In fact, there is a stronger statement.

For primes p5 p \geq 5, (papb)(ab)(modp3) { pa \choose pb } \equiv { a \choose b } \pmod{p^3} .

Calvin Lin Staff - 3 years, 8 months ago

Log in to reply

Sir how do we find the range of p for (papb)(ab)modp4\large{{ pa \choose pb} \equiv {a \choose b} \mod p^4} ?

Md Zuhair - 3 years, 8 months ago

Log in to reply

@Md Zuhair I doubt that holds for a large range.

The p3 p^3 case is also known as wolstenholme's theorem, and the ideas have been used in several olympiad problems.

Calvin Lin Staff - 3 years, 8 months ago

Log in to reply

@Calvin Lin For the need of Olympiads only i am learning this.. RMO is this Sunday. :)

Wish a best of luck for me.

You were one of those people who represented the your motherland. Proud of you :)

Md Zuhair - 3 years, 8 months ago

@Md Zuhair @Rahil Sehgal How may questions did you solve in RMO 2017 ???

Aaron Jerry Ninan - 3 years, 8 months ago

Log in to reply

U first... hw many?

Md Zuhair - 3 years, 8 months ago

Log in to reply

@Md Zuhair I solved 4, but I didnt get the time to write 1 question completely....so effectively I did 3.5... What abt you??

Aaron Jerry Ninan - 3 years, 8 months ago

Log in to reply

@Aaron Jerry Ninan Did 3. But unsure

Md Zuhair - 3 years, 8 months ago

Log in to reply

@Md Zuhair Which ones?

Aaron Jerry Ninan - 3 years, 8 months ago

Log in to reply

@Aaron Jerry Ninan Inequality, Number theory and Combinatorics

Md Zuhair - 3 years, 8 months ago
×

Problem Loading...

Note Loading...

Set Loading...